How Do You Apply the Chain Rule in This Multivariable Calculus Problem?

In summary: If we apply (1) for ##x=tx,y=ty,z=tz## we get that ##f(tx,ty,tz)=(tx)^nf(1,\frac{ty}{tx},\frac{tz}{tx})=t^nx^nf(1,y/x,z/x)##
  • #1
dakota1234
3
1
Moved from a technical forum, so homework template missing
I've been working on this one for a little bit, and I know I really just need to use the chain rule to solve it, but I can't seem to figure out how to set it up properly. Probably a dumb question, but I could really use some help on this!
Screen Shot 2018-11-26 at 4.57.55 PM.png
 

Attachments

  • Screen Shot 2018-11-26 at 4.57.55 PM.png
    Screen Shot 2018-11-26 at 4.57.55 PM.png
    10.4 KB · Views: 539
Physics news on Phys.org
  • #2
I was able to work out a more generalized version of the proof, but I'm not sure how to apply it
Screen Shot 2018-11-26 at 5.03.55 PM.png
 

Attachments

  • Screen Shot 2018-11-26 at 5.03.55 PM.png
    Screen Shot 2018-11-26 at 5.03.55 PM.png
    84.7 KB · Views: 358
  • Like
Likes Delta2
  • #3
you are given that ##f(x,y,z)=x^nf(1,y/x,z/x)## (1). If from (1) you can prove that ##f(tx,ty,tz)=t^nf(x,y,z)## (2) then together with the work already done by you , you ll be finished.

Now if you apply (1) for ##x=tx,y=ty,z=tz## what do you get? if you apply it correctly you ll get a result that if you apply (1) again (as it is directly, without substitution) you ll be able to prove (2).
 
Last edited:
  • #4
Delta2 said:
you are given that ##f(x,y,z)=x^nf(1,y/x,z/x)## (1). If from (1) you can prove that ##f(tx,ty,tz)=t^nf(x,y,z)## (2) then together with the work already done by you , you ll be finished.

Now if you apply (1) for ##x=tx,y=ty,z=tz## what do you get? if you apply it correctly you ll get a result that if you apply (1) again (as it is directly, without substitution) you ll be able to prove (2).
I may just be really tired, but can you explain in more detail how "to apply it correctly"? I've tried plugging in the substitutions for x,y,z, since before I even posted on this forum, and I don't understand what I'm doing wrong.
 
  • #5
Ok, let's say that ##f(u,v,w)=u^nf(1,v/u,w/u)## (1)

Applying (1) for ##u=tx,v=ty,w=tz## we get that ##f(tx,ty,tz)=(tx)^nf(1,\frac{ty}{tx},\frac{tz}{tx})=t^nx^nf(1,y/x,z/x)##

Now i have helped too much i think you must be able to see the last step, what is ##x^nf(1,y/x,z/x)## equal to?
 

1. What is a calculus III proof?

A calculus III proof is a mathematical argument used to prove the validity of a statement or theorem in multivariable calculus. It involves using the principles and techniques of calculus to logically demonstrate the truth of a given statement.

2. Why do I need help with a calculus III proof?

Calculus III involves advanced concepts and techniques such as partial derivatives, multiple integrals, and vector calculus. These can be challenging to understand and apply, so seeking help with a proof can aid in better understanding and mastering these concepts.

3. How do I approach solving a calculus III proof?

Start by carefully reading and understanding the given statement or theorem. Then, analyze the problem and determine which principles and techniques of calculus can be applied. Use clear and logical steps to prove the statement and provide justifications for each step.

4. What are some common mistakes to avoid in a calculus III proof?

Some common mistakes to avoid include making assumptions without justification, skipping steps, and using incorrect mathematical notation. It is also important to check for algebraic errors and ensure that the final result is consistent with the given statement.

5. Where can I find additional resources for help with calculus III proofs?

There are many online resources available such as video tutorials, practice problems, and forums where you can ask for help and clarification. Your textbook and class notes can also be valuable resources. Additionally, seeking help from a tutor or your professor can provide personalized assistance.

Similar threads

  • Calculus and Beyond Homework Help
Replies
1
Views
703
Replies
2
Views
1K
  • Calculus and Beyond Homework Help
Replies
3
Views
1K
  • Calculus and Beyond Homework Help
Replies
4
Views
970
  • Science and Math Textbooks
Replies
9
Views
3K
  • Calculus and Beyond Homework Help
Replies
2
Views
2K
  • Calculus and Beyond Homework Help
Replies
4
Views
1K
Replies
4
Views
1K
  • Calculus and Beyond Homework Help
Replies
2
Views
1K
  • Calculus and Beyond Homework Help
Replies
3
Views
1K
Back
Top